Đến nội dung

Hình ảnh

Topic BẤT ĐẲNG THỨC ôn thi vào lớp 10 THPT 2017 - 2018

bất đẳng thức am-gm cauchy bunyakovski minskovski schwarz holder thcs

  • Please log in to reply
Chủ đề này có 299 trả lời

#161
Mr Cooper

Mr Cooper

    Sĩ quan

  • Thành viên
  • 496 Bài viết

Bài Toán 63. Cho $a,b,c$ là các số thực không âm. Chứng minh rằng:

\[(a+b+c)^3 \ge 6\sqrt{3}(a-b)(b-c)(c-a)\]



#162
Hoang Dinh Nhat

Hoang Dinh Nhat

    Sĩ quan

  • Thành viên
  • 402 Bài viết

​Bài toán 64:(sưu tầm) ​Cho $a,b,c>0$ thỏa mãn $a^3+b^3+c^3=a^4+b^4+c^4$. Chứng minh rằng:

$\frac{a}{a^2+b^3+c^3}+\frac{b}{b^2+c^3+a^3}+\frac{c}{c^2+a^3+b^3}\geq 1$


Chấp nhận giới hạn của bản thân, nhưng đừng bao giờ bỏ cuộc

 

 

 

 


#163
AnhTran2911

AnhTran2911

    Thượng sĩ

  • Thành viên
  • 230 Bài viết

​Bài toán 64:(sưu tầm) ​Cho $a,b,c>0$ thỏa mãn $a^3+b^3+c^3=a^4+b^4+c^4$. Chứng minh rằng:

$\frac{a}{a^2+b^3+c^3}+\frac{b}{b^2+c^3+a^3}+\frac{c}{c^2+a^3+b^3}\geq 1$

cái đề nhìn là biết sai rồi. chỗ thì $a^3$ chỗ thì $a^2$


        AQ02

                                 


#164
Hoang Dinh Nhat

Hoang Dinh Nhat

    Sĩ quan

  • Thành viên
  • 402 Bài viết

cái đề nhìn là biết sai rồi. chỗ thì $a^3$ chỗ thì $a^2$

 

đề không sai nhé bạn


Chấp nhận giới hạn của bản thân, nhưng đừng bao giờ bỏ cuộc

 

 

 

 


#165
AnhTran2911

AnhTran2911

    Thượng sĩ

  • Thành viên
  • 230 Bài viết

Bài Toán 63. Cho $a,b,c$ là các số thực không âm. Chứng minh rằng:

\[(a+b+c)^3 \ge 6\sqrt{3}(a-b)(b-c)(c-a)\]

BP 2 vế và giả sử c min

Ta có $(a-c)^2\leq a^2$ và $(b-c)^2\leq{b^2}$

Quy CM $(a+b+c)^6\geq27. 2ab. 2ab. (a-b)^2$

Theo AM-GM thì $27.2ab.2ab.(a-b)^2\leq(a+b)^6\leq(a+b+c)^6$ ĐPCM.


        AQ02

                                 


#166
AnhTran2911

AnhTran2911

    Thượng sĩ

  • Thành viên
  • 230 Bài viết

​Bài toán 64:(sưu tầm) ​Cho $a,b,c>0$ thỏa mãn $a^3+b^3+c^3=a^4+b^4+c^4$. Chứng minh rằng:

$\frac{a}{a^2+b^3+c^3}+\frac{b}{b^2+c^3+a^3}+\frac{c}{c^2+a^3+b^3}\geq 1$

Theo CS thì:

$LHS\geq\frac{(a+b+c)^2}{\sum{a^3}+\sum{ab(a^2+b^2)}}$=$\frac{(a+b+c)^2}{\sum{a^4}+\sum{ab(a^2+b^2)}}$

=$\frac{a+b+c}{a^3+b^3+c^3}$

Cần CM $a+b+c\geq{a^3+b^3+c^3}$

Hay $(a+b+c)(a^4+b^4+c^4)^2\geq(a^3+b^3+c^3)^3$  Đúng theo HOLDER.


        AQ02

                                 


#167
sharker

sharker

    Sĩ quan

  • Thành viên
  • 301 Bài viết

Bài 65:chứng minh với mọi số thực ta có BDT: $\frac{x^2}{(y-z)^2}+\frac{y^2}{(x-z)^2}+\frac{z^2}{(x-y)^2}\geq 2$


Anh sẽ vẫn bên em dù bất cứ nơi đâu

Anh sẽ là hạt bụi bay theo gió

Anh sẽ là ngôi sao trên bầu trời phương Bắc

Anh không bao giờ dừng lại ở một nơi nào

Anh sẽ là ngọn gió thổi qua các ngọn cây

Em sẽ mãi mãi đợi anh chứ ??

will you wait for me forever


#168
Hoang Dinh Nhat

Hoang Dinh Nhat

    Sĩ quan

  • Thành viên
  • 402 Bài viết

Bài 65:chứng minh với mọi số thực ta có BDT: $\frac{x^2}{(y-z)^2}+\frac{y^2}{(x-z)^2}+\frac{z^2}{(x-y)^2}\geq 2$

 

BĐT cần chứng minh $\Leftrightarrow \frac{(x^3-x^2y-x^2z-xy^2+3xyz-xz^2+y^3-y^2z-yz^2+z^3)^2}{(y-x)^2)(z-x)^2(z-y)^2}$$\geq 0$

BĐT cuối luôn đúng nên BĐT được chứng minh


Bài viết đã được chỉnh sửa nội dung bởi Hoang Dinh Nhat: 13-05-2017 - 19:58

Chấp nhận giới hạn của bản thân, nhưng đừng bao giờ bỏ cuộc

 

 

 

 


#169
Doflamingo

Doflamingo

    Hạ sĩ

  • Thành viên
  • 82 Bài viết

BĐT cần chứng minh $\Leftrightarrow \frac{(x^3-x^2y-x^2z-xy^2+3xyz-xz^2+y^3-y^2z-yz^2+z^3)^2}{(y-x)^2)(z-x)^2(z-y)^2}$$\geq 0$

BĐT cuối luôn đúng nên BĐT được chứng minh

biến đổi tương đương kiểu này thì chắc siêu rắc rối luôn



#170
Doflamingo

Doflamingo

    Hạ sĩ

  • Thành viên
  • 82 Bài viết

Bài toán 66(sưu tầm)

Cho các số dương x,y,z thỏa mãn $x^{2}+y^{2}+z^{2}\geqslant 3$. CMR:

$\frac{x^{5}-x^{2}}{x^{5}+y^{2}+z^{2}}+\frac{y^{5}-y^{2}}{y^{5}+z^{2}+x^{2}}+\frac{z^{5}-z^{2}}{z^{5}+x^{2}+y^{2}}\geqslant 0$


Bài viết đã được chỉnh sửa nội dung bởi Doflamingo: 14-05-2017 - 00:09


#171
Hoang Dinh Nhat

Hoang Dinh Nhat

    Sĩ quan

  • Thành viên
  • 402 Bài viết

Bài toán 66(sưu tầm)

Cho các số dương x,y,z thỏa mãn $x^{2}+y^{2}+z^{2}\geqslant 3$. CMR:

$\frac{x^{5}-x^{2}}{x^{5}+y^{2}+z^{2}}+\frac{y^{5}-y^{2}}{y^{5}+z^{2}+x^{2}}+\frac{z^{5}-z^{2}}{z^{5}+x^{2}+y^{2}}\geqslant 0$

 

Bất đẳng thức cần chứng minh tương đương với $\sum \frac{1}{x^5+y^2+z^2}\leq \frac{3}{x^2+y^2+z^2}$

Từ đây ta chỉ cần xét trương hợp: $x^2+y^2+z^2=3$ nên bất đẳng thức cần chứng minh trở thành:

$\sum \frac{1}{x^5-x^2+3}\leq 1$

Theo AM-GM, ta có: $x^5=\frac{x^6}{x}\geq \frac{2x^6}{x^2+1}$

Đặt $a=x^2;b=y^2;c=z^2\Rightarrow a+b+c=3$ Bất đẳng thức cần chứng minh trở thành

$\sum \frac{1}{\frac{2a^3}{a+1}-a+3}\leq 1\Leftrightarrow \sum \frac{a+1}{2a^3-a^2+2a+3}\leq 1\Leftrightarrow \frac{(a-1)^2(-2a^3+3a+3)}{2a^3-a^2+2a+3}\geq 0$ (1)

Không mất tính tổng quát giả sử: $a\geq b\geq c\Rightarrow a\geq 1\geq c$. Xét hai trường hợp:

TH1: $b+c\geq 1\Rightarrow a\leq 2$, lúc đó:

$-2a^3+3a+3>0;-2b^3+3b+3>0;-2c^3+3c+3>0$ nên (1) đúng

TH2: $b+c\leq 1\Rightarrow a\geq 2$, lúc đó:

$(2a^3-a^2+2a+3)-5(a+1)=2a^3-a^2-3a-2=a^3(2-\frac{1}{a}-\frac{3}{a^2}-\frac{2}{a^3})\geq a^3(2-\frac{1}{2}-\frac{3}{2^2}-\frac{2}{2^3})=\frac{a^3}{2}>0$

$\Rightarrow \frac{a+1}{2a^3-a^2+2a+3}\leq \frac{1}{5}$. Cần chứng minh:

$\frac{b+1}{2b^3-b^2+2b+3}+\frac{c+1}{2c^3-c^2+2c+3}\leq \frac{4}{5}$

Ta có bổ đề: Với mọi $0<x<1$, ta có: $\frac{x+1}{2x^3-x^2+2x+3}\leq \frac{2}{5}\Leftrightarrow 4x^3\geq (x+1)(2x-1)$

+TH1: Nếu $x\leq \frac{1}{2}$, ta có điều phải chứng minh

+TH2: Nếu $x\geq \frac{1}{2}$ ta có:

$4x^3-(x+1)(2x-1)\geq 4x^3-2(2x-1)=2(2x^3-2x+1)\geq 2(x^2-2x+1)=2(x-1)^2\geq 0$ (đúng)

Ta có điều cần chứng minh

Đạt tại: $a=b=c=1$


Chấp nhận giới hạn của bản thân, nhưng đừng bao giờ bỏ cuộc

 

 

 

 


#172
Doflamingo

Doflamingo

    Hạ sĩ

  • Thành viên
  • 82 Bài viết

còn bài 59,61 chưa giải kìa



#173
Nguyenhuyen_AG

Nguyenhuyen_AG

    Trung úy

  • Thành viên nổi bật 2016
  • 945 Bài viết

Bài toán 61(sưu tầm)

Cho a,b,c là các số thực không âm, đôi một khác nhau. CMR:

$(ab+bc+ca)\left [ \frac{1}{(a-b)^{2}}+\frac{1}{(b-c)^{2}}+\frac{1}{(c-a)^{2}} \right ]\geqslant 4$

 

Ta có

\[\text{Vế trái  -Vế phải} = \sum \frac{ab\left[c(a+b-c)+a^2-3ab+b^2\right]^2}{(a-b)^2(b-c)^2(c-a)^2} \geqslant 0.\]

P/s. Anh nhớ Doflamingo bị Hải quân bắt rồi sao còn ở đây post bài nhỉ?


Nguyen Van Huyen
Ho Chi Minh City University Of Transport

#174
Doflamingo

Doflamingo

    Hạ sĩ

  • Thành viên
  • 82 Bài viết

Bài toán 67
Untitled.png

f.png

h.png


Bài viết đã được chỉnh sửa nội dung bởi Doflamingo: 14-05-2017 - 16:42


#175
tuaneee111

tuaneee111

    Trung sĩ

  • Thành viên
  • 174 Bài viết

Bài 2: 

Ta có: $$\sum\limits_{cyc} {\frac{{{{\left( {\sqrt b } \right)}^2} + {{\left( {\sqrt c } \right)}^2}}}{{\sqrt a }}}  + \sum\limits_{cyc} {2\sqrt a }  = \sum\limits_{cyc} {\left[ {\frac{{{{\left( {\sqrt b } \right)}^2} + {{\left( {\sqrt c } \right)}^2}}}{{\sqrt a }} + 2\sqrt a } \right]}  \geqslant \sum\limits_{cyc} {2\sqrt {2\left( {{{\left( {\sqrt b } \right)}^2} + {{\left( {\sqrt c } \right)}^2}} \right)} }  \geqslant \sum\limits_{cyc} {2\left( {\sqrt b  + \sqrt c } \right)} $$

Vậy cần chứng minh $\sqrt a  + \sqrt b  + \sqrt c  \geqslant 3$. Điểu này luôn đúng theo $AM-GM$ nên có điều phải chứng minh!


Bài viết đã được chỉnh sửa nội dung bởi tuaneee111: 14-05-2017 - 17:12

$$\boxed{\boxed{I\heartsuit MATHEMATICAL}}$$

Blog của tôi

:luoi: Sức hấp dẫn của toán học mãnh liệt đến nỗi tôi bắt đầu sao nhãng các môn học khác - Sofia Vasilyevna Kovalevskaya :lol:


#176
HoangKhanh2002

HoangKhanh2002

    Sĩ quan

  • Thành viên
  • 483 Bài viết

Bài 3:

BĐT đã cho tương đương với: $2(\sum \sqrt{a})\geqslant 2\sum ab\Leftrightarrow 2(\sum \sqrt{a}) \geqslant (\sum a)^2-(\sum a^2) \Leftrightarrow \sum a^2+2(\sum a) \geqslant 9$

Mặt khác, theo BĐT $AM-GM$ ta có: $a^2+\sqrt{a}+\sqrt{a}\geqslant 3a\Rightarrow \sum a^2+2(\sum \sqrt{a})\geqslant 3(\sum a)=9\Rightarrow Q.E.D$

Bài 7:

Ta có: $(a+b)(b+c)(c+a)+4=(a+b+c)(ab+bc+ca)+3=\dfrac{(a+b+c)(ab+bc+ca)}{3}+\dfrac{(a+b+c)(ab+bc+ca)}{3}+\dfrac{(a+b+c)(ab+bc+ca)}{3}+3\geqslant 4\sqrt[4]{\dfrac{(a+b+c)^3(ab+bc+ca)^3}{9}}= 4(a+b+c)\sqrt[4]{\dfrac{(ab+bc+ca)^3}{9(a+b+c)}}$

Mà: $9(a+b+c)=9abc(a+b+c)\leqslant 3(ab+bc+ca)^2\leqslant (ab+bc+ca)^3$ (vì $ab+bc+ca\geqslant 3\sqrt[3]{a^2b^2c^2}=3$

Do đó ta có đpcm


Bài viết đã được chỉnh sửa nội dung bởi HoangKhanh2002: 14-05-2017 - 17:14


#177
viet9a14124869

viet9a14124869

    Trung úy

  • Thành viên
  • 903 Bài viết

Em thấy bác đăng nhiều bài trong một thế này không hiệu quả lắm ,có khi còn do nhiều bài bị đọng lại ,làm loãng topic :D

Bài toán 67
attachicon.gifUntitled.png

attachicon.giff.png

attachicon.gifh.png

Chém mấy câu dễ trước

Bài 4 , Ta chứng minh bất đẳng thức phụ $a^5-a^2+3\geq a^3+2\Leftrightarrow (a-1)^2(a+1)(a^2+1)\geq 0$ ( luôn đúng )

Do đó ta quy về chứng minh $(a^3+1+1)(1+b^3+1)(1+1+c^3)\geq (a+b+c)^3$ ( đúng theo Holder)

Bài 6 , chắc là biến đổi tương đương hay dùng S.O.S chăng :D

 

p/s : Bài 59 có ai có lời giải không #


                                                                    SÓNG BẮT ĐẦU TỪ GIÓ

                                                                    GIÓ BẮT ĐẦU TỪ ĐÂU ?

                                                                    ANH CŨNG KHÔNG BIẾT NỮA 

                                                                    KHI NÀO...? TA YÊU NHAU .


#178
TrBaoChis

TrBaoChis

    Hạ sĩ

  • Banned
  • 81 Bài viết

Bài toán 67
attachicon.gifUntitled.png

attachicon.giff.png

attachicon.gifh.png

bài 1 dễ ko ai làm à  :D 
$\frac{a}{b}$ + $\frac{a}{b}$ + $\frac{b}{c}$ $\geq$ $3\sqrt[3]{\frac{a^2}{bc}}$ = $3a$ , cộng vtv $\Rightarrow$ ĐPCM


Bài viết đã được chỉnh sửa nội dung bởi TrBaoChis: 14-05-2017 - 18:23


#179
Nguyenphuctang

Nguyenphuctang

    Sĩ quan

  • Banned
  • 499 Bài viết

P/s. Anh nhớ Doflamingo bị Hải quân bắt rồi sao còn ở đây post bài nhỉ?

Anh tiễn ra đảo hộ em với, lần nào report cũng khá lâu :3 mới ra đảo :3
 

Bài toán đề xuất tiếp theo:

Bài 68: 

Cho a,b,c là số thực không âm, thỏa mãn $a+b>0$, $b+c>0$,$c+a>0$. Chứng minh rằng:

$$\sqrt {\frac{a}{{b + c}}}  + \sqrt {\frac{b}{{a + c}}}  + \sqrt {\frac{c}{{a + b}}}  + \frac{{9\sqrt {ab + bc + ac} }}{{a + b + c}} \ge 6$$


Bài viết đã được chỉnh sửa nội dung bởi Nguyenphuctang: 14-05-2017 - 19:02


#180
viet9a14124869

viet9a14124869

    Trung úy

  • Thành viên
  • 903 Bài viết

Anh tiễn ra đảo hộ em với, lần nào report cũng khá lâu :3 mới ra đảo :3
 

Bài toán đề xuất tiếp theo:

Bài 68: 

Cho a,b,c là số thực không âm, thỏa mãn $a+b>0$, $b+c>0$,$c+a>0$. Chứng minh rằng:

$$\sqrt {\frac{a}{{b + c}}}  + \sqrt {\frac{b}{{a + c}}}  + \sqrt {\frac{c}{{a + b}}}  + \frac{{9\sqrt {ab + bc + ac} }}{{a + b + c}} \ge 6$$

Bài này dạo trước chị trambau có giải rồi

Bài 68 ,

Sử dúng bất đẳng thức Holder

$\sum \sqrt{\frac{a}{b+c}}\geq \sqrt{\frac{(a+b+c)^3}{\sum a^2(b+c)}}\geq \sqrt{\frac{(a+b+c)^3}{(a+b+c)(ab+bc+ca)}}=\frac{a+b+c}{\sqrt{ab+bc+ca}}$

Vậy bất đẳng thức cần chứng minh trở thành $\frac{a+b+c}{\sqrt{ab+bc+ca}}+\frac{9\sqrt{ab+bc+ca}}{a+b+c}\geq 6$ ( đúng theo AM-GM )4

Dấu bằng xẩy ra khi a=0 , $b=\frac{7\pm 3\sqrt{5}}{2}c$ và các hoán vị


                                                                    SÓNG BẮT ĐẦU TỪ GIÓ

                                                                    GIÓ BẮT ĐẦU TỪ ĐÂU ?

                                                                    ANH CŨNG KHÔNG BIẾT NỮA 

                                                                    KHI NÀO...? TA YÊU NHAU .






Được gắn nhãn với một hoặc nhiều trong số những từ khóa sau: bất đẳng thức, am-gm, cauchy, bunyakovski, minskovski, schwarz, holder, thcs

1 người đang xem chủ đề

0 thành viên, 1 khách, 0 thành viên ẩn danh